hello can you help me with this?

Hello Can You Help Me With This?

Answers

Answer 1

Answer:

x = 25.5

Step-by-step explanation:

suppose RS and MQ are parallel the sum of angle MRS and RMN would be 180 degrees

2x + x + x 78 = 180 add like terms then subtract 78 from both sides

4x = 102 divide both sides by 4

x = 25.5


Related Questions

It would be great if someone could help me with this problem

Answers

y-intercept (0, 1)
Equation of asymptote y=0

Equation of asymptote can also be called horizontal asymptote.
It means the place where the line almost meets but actually not touching the place. The line almost touches y=0 but it actually does not touch it.

What is the answer to this question

Answers

26 m rhiendjfnrbsjen

Answer:

26 m is the answer to the question

The two
triangles are similar.

What is the value of x?

Enter your answer in the box

Answers

Answer:

13

Step-by-step explanation:

the scale factor is 4:1 so just divide 52 by 4.

Calculate the exact value of number one

Answers

Answer:

3/ 20

Step-by-step explanation:

4 1/5 × 1/3 - 1 1/4

= 21/5 × 1/3 - 1/5

= 7/5 - 5/4 = 28 - 25/ 20 = 3/20

= 3/20

Solve the system of equations.
5x – 4y = -10
y = 2 x – 5
x =
y =

Answers

Answer:

6x2+3y2=12,x+y=2,2x-y=-1x+y=4,x-y=2

Step-by-step explanation:

I just know.

Answer:

Step-by-step explanation:

5x – 4y = -10

y = 2 x – 5

Rearrage the equation:

5x – 4y = -10 (1)

-2 x +y =  5.  (2)

pluging variable y equation2 in 1

5x - 4•(2x-5) = -10

 -3x = -30

Solve equation [1] for the variable  x  

3x = 30

x = 10

x = 10

y = 2x-5

Use the  x  value to solve for  y  

y = 2(10)-5 = 15

x= 10

y= 15

Help me out plz bbbhbbhh

Answers

5.) -3x - 12
hopefully that’s the answer you needed

the solution is in the picture ☝️

ANSWER: 240 hours

this is for n.12

The table below shows how much Joe earns, y, after working x hours.

Joe’s Earnings

Hours worked
Money earned
4
$30
10
$75
12
$90
22
$165

The relationship between money earned and hours worked is linear. Joe computes the slope between (4, 30) and (12, 90), then computes the slope between (4, 30) and (10, 75). How do the two slopes compare?
The slope between (4, 30) and (12, 90) is greater because the ordered pairs are farther apart on the x-axis.
The slope between (4, 30) and (12, 90) is greater because the ordered pairs are farther apart on the y-axis.
The slope between (4, 30) and (12, 90) and between (4, 30) and (10, 75) is the same.
The slope between (4, 30) and (12, 90) is less because 4 is a factor of 12 and 30 is a factor of 90.

Answers

Answer: Given : Joe’s Earnings and hour worked

The relationship between money earned and hours worked is linear.

Joe computes the slope between (4, 30) and (12, 90), then computes the slope between (4, 30) and (10, 75).

To Find : How do the two slopes compare?

Solution:

Hours worked     Money earned

4                          $30

10                        $75

12                        $90

22                       $165

slope between (4, 30) and (12, 90),

= (90 - 30)/(12 - 4)

= 60/8

= 15/2

slope between (4, 30) and (10, 75)

= (75 - 30)/(10-4)

= 45/6

= 15/2

The slope between (4, 30) and (12, 90) and between (4, 30) and (10, 75) is the same.

Both Slopes are same.

i hope this helped and have a nice day/night

The slope between (4, 30) and (12, 90) and between (4, 30) and (10, 75) is the same. The correct answer is the third option.

The slope between two points (x₁,y₁)  and  (x₂, y₂) on a straight line is given by (y₂ - y₁)/(x₂ -x₁ ).

Let's calculate the slopes:

The slope between (4, 30) and (12, 90):

Slope = (90 - 30) / (12 - 4)

= 60 / 8

= 7.5

The slope between (4, 30) and (10, 75):

Slope = (75 - 30) / (10 - 4)

= 45 / 6

= 7.5

As we can see, the slopes between the two sets of ordered pairs are the same.

Thus,  the slope between (4, 30) and (12, 90) and between (4, 30) and (10, 75) is the same.

Hence, the correct answer is the third option.

Learn more about the slope of the line here:

brainly.com/question/14511992

#SPJ7


please help me !! ! !

Answers

option a. option a has an answer of x=13, while the other problems have solutions of x=5.


Help and explain pls and thankyouu

Answers

Hello,

1)

[tex]\left\{\begin{array}{ccc}2x-y&=&7\\-2x+3y&=&9\\\end{array} \right.\\\\\\\left\{\begin{array}{ccc}2y&=&16\\-2x+3y&=&9\\\end{array} \right.\\\\\\\left\{\begin{array}{ccc}y&=&8\\-2x+3*8&=&9\\\end{array} \right.\\\\\\\left\{\begin{array}{ccc}x&=&7.5\\y&=&8\\\end{array} \right.\\\\[/tex]

Answer C

2)

[tex]\left\{\begin{array}{ccc}x-2y-3z&=&5\\x+2y+3z&=&7\\x+z&=&3\\\end{array} \right.\\\\\\\left\{\begin{array}{ccc}2x&=&12\\x+2y+3z&=&7\\x+z&=&3\\\end{array} \right.\\\\\\\left\{\begin{array}{ccc}x&=&6\\2y+3z&=&1\\z&=&-3\\\end{array} \right.\\\\\\\\\left\{\begin{array}{ccc}x&=&6\\y&=&5\\z&=&-3\\\end{array} \right.\\\\\\[/tex]

Answer C

What is the result of converting 60 ounces to punds remember there are 16 ounces in a pound pleasdnsjjsjs

Answers

Answer:

A. 3.75 pounds

Step-by-step explanation:

16 ounces = 1 pound

converting 60 ounces to pounds

Let x = number of pounds

60 ounces = x pounds

Find the proportion

16 / 1 = 60 / x

Cross product

16 * x = 1 * 60

16x = 60

Divide both sides by 16

x = 60/16

x = 3.75

Therefore,

60 ounces = 3.75 pounds

What statements are true about parallegram?

Answers

I think it’s the 2nd third and fifth but I’m just answering this so I wouldn’t Answer it off my answer
2nd 3rd and 5th is the answer to this

A positive integer is multiply by 5, then 6 is subtracted from the result, and then the last result is
divided by 8. (figure it out)

Answers

Answer:

3.75

Step-by-step explanation:

5 times 6  30/8

Suppose you had been in charge of designing the study. What sample size would be needed to construct a margin of error of 2% with 95% confidence

Answers

Answer:

A sample size of 2401 is needed.

Step-by-step explanation:

In a sample with a number n of people surveyed with a probability of a success of [tex]\pi[/tex], and a confidence level of [tex]1-\alpha[/tex], we have the following confidence interval of proportions.

[tex]\pi \pm z\sqrt{\frac{\pi(1-\pi)}{n}}[/tex]

In which

z is the z-score that has a p-value of [tex]1 - \frac{\alpha}{2}[/tex].

The margin of error is given by:

[tex]M = z\sqrt{\frac{\pi(1-\pi)}{n}}[/tex]

95% confidence level

So [tex]\alpha = 0.05[/tex], z is the value of Z that has a p-value of [tex]1 - \frac{0.05}{2} = 0.975[/tex], so [tex]Z = 1.96[/tex].

What sample size would be needed to construct a margin of error of 2% with 95% confidence?

This is M for which n = 0.02.

Supposing we have no estimate for the true proportion, we use [tex]\pi = 0.5[/tex].

[tex]M = z\sqrt{\frac{\pi(1-\pi)}{n}}[/tex]

[tex]0.02 = 1.96\sqrt{\frac{0.5*0.5}{n}}[/tex]

[tex]0.02\sqrt{n} = 1.96*0.5[/tex]

[tex]\sqrt{n} = \frac{1.96*0.5}{0.02}[/tex]

[tex](\sqrt{n})^2 = (\frac{1.96*0.5}{0.02})^2[/tex]

[tex]n = 2401[/tex]

A sample size of 2401 is needed.

all of the following equations are equivalent except? -5 (x - 1 ), -5x + 5, 5 - 5x, (5-5) x

Answers

Except the last one (5-5)x

A monthly budget with expenses is shown.

Monthly Budget for a Family of Three
in San Antonio, Texas
Category Amount of Money
Rent $1,200
Food $800
Utilities $400
Gasoline $320
Auto insurance $250
Health insurance $420
Medical expenses $300
Childcare $360
Savings $300
Other $150
Problem

Which equation can be used to determine y, the minimum amount of money a family must earn to meet the requirements of this budget for one year?

Answers

It will be 52200 on the dot you walk come

What should you substitute for y in the bottom equation to solve the system by the substitution method?

A. y=3x+15

B. y =-x-5

C. y=x+5

D. y=-3-15

Answers

Answer: y= -x-5


Step 1: Determine which equation to change

Since we will be using the substitution method, we will need to substitute y from the bottom equation with information from the top equation. This will be the equation we change.

Step 2: Change the equation

Let’s rewrite the equation we will be working with.

3x+3y= -15

Our goal will be to get y alone on the left side. To start, let’s subtract 3x from both sides, leaving only 3y on the left side.

3x+3y= -15
3y= -3x-15

Now we need to get y completely alone by eliminating the coefficient of 3. Let’s do this by dividing each term by 3.

3y= -3x-15
y= -x-5


This is your answer. Hope this helps! Comment below for more questions.

What is the true solution to the logarithmic equation below?

log Subscript 4 Baseline left-bracket log Subscript 4 Baseline (2 x) right-bracket = 1

Answers

Answer:

x = 2

Step-by-step explanation:

Given:

log ₄ (2x) = 1

Solving for x:

2x = 4¹2x = 4x = 2

Answer:

128 on edg

Step-by-step explanation:

i thought it was 2 aswell double checked it on a graphing calculator its 128

Which congruence theorem can be used to prove BDAS DBC?​

Answers

Answer:

We know that the hypotenuse-leg theorem states that if the hypotenuse and one leg of a right triangle are congruent to hypotenuse and corresponding leg of another right triangle, then the triangles are congruent. hypotenuse(AB) of △BDA equals to hypotenuse (CD) of △DBC.

Answer:

A. HL

Step-by-step explanation:

A farmer builds a fence to enclose a rectangular pasture. He uses 155 feet of fence. Find the total area of the pasture if it is 45.5 feet long. The length is 6 and the height is 5 what is the width?

Answers

Answer: [tex]1456\ ft^2[/tex]

Step-by-step explanation:

Given

Length of whole fence is 155 feet

If the length of rectangle is 45.5 ft

Suppose width is w

Length of whole fence is perimeter which is given by

[tex]\Rightarrow 155=2(45.5+w)\\\Rightarrow 77.5=45.5+w\\\Rightarrow w=32\ ft[/tex]

Area of the rectangle is given by the product of length and width

[tex]\Rightarrow A=lw\\\Rightarrow A=45.5\times 32\\\Rightarrow A=1456\ ft^2[/tex]

Thus, total area of pasture is [tex]1456\ ft^2[/tex]

What fraction of 300 gallons is 117 L? Give your answer in simplest form. 1 gallon = 4.5 L.

Answers

Answer:

Use proportions to find the gallon equivalent of 117 L:

[tex]\frac{1gal}{4.5L} =\frac{Xgal}{117L}[/tex]

Cross-multiply & solve:

[tex]4.5x=117\\x=26[/tex]

So 117 L = 26 gallons

Out of 300 gallons, it's

[tex]\frac{26}{300} =\frac{2(13)}{2(150)} =\frac{13}{150}[/tex]

I hope this is how you're suppose to solve it..

Greg buys 60 garden plants at a cost price of $2.00 each to sell in his shop. He sells 25 of them at the profit of 75% and 18 of them at the profit of 35%. He sells the rest of the plants for 4/5 of the cost price calculate the profit or loss he makes from selling 60 plants stating if it is a profit or loss

Answers

Answer:

$43.30 profit

Step-by-step explanation:

Total cost of plant:

60*2 = 120

Greg makes total of:

25*(2 + 0.75*2) + 18*(2 + 0.35*2) + (60 - 25 - 18)*2*4/5 = 163.3

Since Greg mare than cost, he has a profit and the amount is:

163.3 - 120 = 43.3

Which point is in the solution set of this system inequalities?

A. (0,0)

B. None of these

C. (5,1)

D. (3,7)

Answers

Answer:

B

Step-by-step explanation:

To find which ordered pairs are solutions to the inequalities we can simply plug in the x and y values of the ordered pairs into the inequalities and if the equation is true for both inequalities then the ordered pair is a solution to the inequalities.

For (0,0)

x = 0

y = 0

y > x + 5

Substitute 0 for y and x

0 > 0 + 5

Simplify right side

0 > 5

The inequality is not true as 5 is greater than 0, not less than. So immediately we can eliminate answer choice A.

For (5,1).

x = 5

y = 1

y > x + 5

Substitute 5 for x and 1 for y

1 > 5 + 5

Simplify right side

1 > 10

Again, the equation is not true as 1 is not greater than 10. This means that c cannot be the answer

For (3,7)

x = 3

y = 7

y > x + 5

Substitute 3 for x and y for 7

7 > 3 + 5

Simplify right side

7 > 8

7 is not greater than 8 meaning that (3,7) cannot be a solution to the inequalities

None of the ordered pairs created true equations hence the answer is B

Write a recursive rule for the sequence.
x, x, 2x, 3x, 5x, 8x, ...
I know that it adds its last term but I don't know the rule/formula to show that.

Answers

Answer:

f(n)=f(n-1)+f(n-2)

f(1)=1x

f(2)=1x

Step-by-step explanation:

This is the fibonacci sequence with each term times x.

Notice, you are adding the previous two terms to get the third term per consecutive triples of the sequence.

That is:

1x+1x=2x

1x+2x=3x

2x+3x=5x

3x+5x=8x

So since we need the two terms before the third per each consecutive triple in the sequence, our recursive definition must include two terms of the sequence. People normally go with the first two.

f(1)=1x since first term of f is 1x

f(2)=1x since second term of f is 1x

Yes, I'm naming the sequence f.

So I said a third term in a consecutive triple of the sequence is equal to the sum of it's two prior terms. Example, f(3)=f(2)+f(1) and f(4)=f(3)+f(2) and so on...

Note, the term before the nth term is the (n-1)th term and the term before the (n-1)th term is the (n-2)th term. Just like before the 15th term you have the (15-1)th term and before that one you have the (15-2)th term. That example simplified means before the 15th term you have the 14th and then the 13th.

So in general f(n)=f(n-1)+f(n-2).

So the full recursive definition is:

f(n)=f(n-1)+f(n-2)

f(1)=1x

f(2)=1x

A self-serve frozen yogurt shop has 8 candy toppings and 4 fruit toppings to choose from. How many ways are there to top a frozen yogurt?

hint.. correct answer is 4096

Answers

4096 ways : 8 candy toppings plus 4 fruit toppings is 12. Then 2^12 (2 different types of toppings and the 12 of total toppings) = 4096

What value of g makes the equation true?

Answers

Answer:

[tex](x + 7)(x - 4) \\ = { \tt{ {x}^{2} + 7x - 4x - 28 }} \\ = { \tt{ {x}^{2} + 3x - 28 }} \\ { \boxed{ \bf{g \: is \: 3}}}[/tex]

Answer:

g=3

Step-by-step explanation:

Hi there!

To find the value of g, we can expand the two binomials using the distributive property:

[tex](x+7)(x-4)\\= x(x-4)+7(x-4)\\= x^2-4x+7x-28\\= x^2+3x-28[/tex]

Therefore, the value of g that makes the equation true is 3.

I hope this helps!

Find the value of n.
PLEASE HELP ASAP!

Answers

Answer:

d .9

Step-by-step explanation:

n^2-3 = 2×39 (line joining midpoints is half of third line )

n^2 = 78+3

n^2 =81

n =9

The width of a rectangle is (2x – 7)inches and its width is (x^2 – 5) inches. Find an expression for the perimeter of the rectangle.
a. 2x^3 + 35
b. x^2 - 2x + 2
c. x^2 + 2x – 12
d. 2x^2 + 4x – 24

Answers

Answer:

(2x²+4x-24) in.

Step-by-step explanation:

.

the answer is D you’re welcome

Plz help. How to convert this standard notation to scientific notation 549,755,813,888.

Answers

Answer:

To change a number from scientific notation to standard form, move the decimal point to the left (if the exponent of ten is a negative number), or to the right (if the exponent is positive). You should move the point as many times as the exponent indicates. Do not write the power of ten anymore

The students in a school can be arranged in 12, 15 and 18 equal rows and also into a solid square. What is the lowest number of students that can be in the school?

Answers

Hello,

Let's x then number of students.

x is divisible by 12,15,18 thus by 180 (lcm)

x=180*k is a square

2²*3²*5*k= a square ==> k=5

x=180*5=900

Proof:

900/12=75

900/15=60

900/18=50

900=30²

please tell me the answer to this, thanks

Answers

Step-by-step explanation:

Hey there!

The equation is: 2x + 3y -12=0 (i)

From the given equation;

Slope (m) = (-coeff.of X)/(coeff.of y)

m = -2/3

Now; Let the missing component be "a".

The equation also can be written in;

The equation also can be written in;(y-8) = -2/3(x-a) (using one point formula)

3y - 24 = -2x + 2a

2x +3y -2a -24 = 0………(ii)

Since both equation are made from same coordinates, equating equation (i) and (ii).

2x + 3y -12= 2x +3y -2a -24

Cancel like terms,

-2a - 24 = -12

or, -2a = 12

or, a = -6.

Therefore, the missing value is -6.

Check:

Putting the value "a" in equation (ii).

2x + 3y -2(-6) -24 = 0

or, 2x+3y -12 = 0 (which is equal to equation (i).

Hope it helps!

Other Questions
How to live in harmony with the Universe without hunting anyone or anything, from ....................1.From a patriotic person2.From Saints and Gurus3.From a hunter A solid box with height 20cm, width 50cm and length 60cm needs to be painted.The paint costs 0.06 per cm2.How much will it cost to paint the box? What does it mean when a Guatemalan shakes his hand quickly so that the index and middle fingers slap together and make a snapping sound?He is trying to get a girl's attention.He is adding emphasis, expressing surprise, or indicating hurry.He is showing affection.He is hungry. The correct and best answer will be marked as brainiest What is the direction of the magnetic field due to the positive charge at the location of the negative charge General Circulation Models (GCM) :_________a) use data collected exclusively from high-resolution satellites. b) use spectral models derived from energy released from the earth and clouds. c) can be run on powerful home computers, allowing citizen scientists to run models. d) use complicated two-dimensional grid systems that change temporally. i will give brainliest The business intelligence environment includes all of the following except: A. BI Infrastructure B. Business Analytics C. Data from the business enviroment D. Cloud-based Storage HELP WILL GIVE BRAINLIEST As was the case in Donatello's David (Fig. 17.21), Michelangelo's version was conceived of as a(n) _____ It was found that ice cream sales decrease significantly as the temperature outside decreases; a very strong relationship was found. What is the most likely correlation coefficient from this hypothetical study Name the following compound. Group of answer choices 2-methyl-4-pentyne 4-methyl-3-propyl-1-pentyne 3-isopropyl-1-hexyne 1-nonyne 4-propyl-5-hexyne A particle charge of 2.7 C is at the center of a Gaussian cube 55 cm on edge. What is the net electric flux through the surface? what is the answer to this Keys, Inc. purchased 100 shares of its own common stock for $10 per share. The stock is now classified as ______ stock, a contra equity account, reported on the statement of stockholder's equity. Which statement best completes the diagram? Match the work of art to he movement/Time period listed1. MegalithsA. Upper Paleolithic B. Mesolithic C. Neolithic Which graph represents the function below? y= { -x if x > -3 x+6, if x helpppppppppppppppppp where is Liberia's location in Africa?